0
$\begingroup$

How can I use the delta method to show that

$\sqrt n$ (1/$\bar Y_n$ - 1/$\mu)$ $\rightarrow$ N (0, $\sigma^2$/$\mu^4$) ?

We know that:

$\bar Y_n$ = $1/n \sum_{n=1}^n Y_i$

$\ Y_i$ is i.i.d; $E(\ Y_i )= \mu\ne 0$ and $V(\ Y_i)= \sigma^2 \gt$ 0

$\endgroup$

1 Answer 1

3
$\begingroup$

Set $f(X) = \frac{1}{X}$ (for $f\colon W\longrightarrow \mathbb R$). Then $Df(X) = -\frac{1}{X^2}$ ($Df$ denotes the derivative). The delta-method states that $$\sqrt n(f(X) - f(\mu)) \rightarrow N(0,\sigma^2Df(\mu)^2)$$ provided that $\sqrt n(X-\mu)\rightarrow N(0,\sigma^2)$. Now your job is to plug in the right value for $X$ and you are done ;-)

$\endgroup$
5
  • $\begingroup$ Is that sufficient as a proof? $\endgroup$ Jan 9, 2017 at 22:34
  • $\begingroup$ I would say yes. $\endgroup$ Jan 11, 2017 at 11:21
  • $\begingroup$ Ok, I was just wondering why you did not apply Slutsky as one of the steps. $\endgroup$ Jan 11, 2017 at 14:06
  • $\begingroup$ It's not necessary because we are allowed to use the delta method (however, delta method requires that $\sqrt n(X-\mu)$ converges in distribution. If this is not given, you have to proof it first of course, but that's a new question) $\endgroup$ Jan 11, 2017 at 14:13
  • $\begingroup$ Ah, ok! I thought I have to show that as well $\endgroup$ Jan 11, 2017 at 14:38

Your Answer

By clicking “Post Your Answer”, you agree to our terms of service and acknowledge you have read our privacy policy.

Not the answer you're looking for? Browse other questions tagged or ask your own question.